Lösung.

Zunächst untersuchen wir die Funktion auf lokale Extrema auf der offenen Menge $ \mbox{$\{(x,y)^\text{t}\in\mathbb{R}^2\,\vert\,x^2+y^2 < 1\}$}$ .

Die notwendige Bedingung

$ \mbox{$\displaystyle
f'(x,y) \;=\; ((y+1)e^{x(y+1)}, x e^{x(y+1)}) \;\overset{\!}{=}\; 0
$}$
liefert
$ \mbox{$\displaystyle
\begin{array}{rcl}
y+1 & = & 0\\
x & = & 0\; .
\end{array}$}$
Der einzige kritische Punkt ist somit der Punkt $ \mbox{$(0,-1)^\text{t}$}$ , der aber nicht in der Menge $ \mbox{$\{(x,y)^\text{t}\in\mathbb{R}^2\,\vert\,x^2+y^2 < 1\}$}$ liegt.

Also gibt es auf $ \mbox{$\{(x,y)^\text{t}\in\mathbb{R}^2\,\vert\,x^2+y^2 < 1\}$}$ kein lokales Extremum. Eine globale Extremstelle auf der gesamten Menge $ \mbox{$\{(x,y)^\text{t}\in\mathbb{R}^2\,\vert\,x^2+y^2 \leq 1\}$}$ , welche in $ \mbox{$\{(x,y)^\text{t}\in\mathbb{R}^2\,\vert\,x^2+y^2 < 1\}$}$ liegt, wäre aber insbesondere eine solche lokale Extremstelle. Damit gibt es auf $ \mbox{$\{(x,y)^\text{t}\in\mathbb{R}^2\,\vert\,x^2+y^2 < 1\}$}$ auch kein globales Extremum.

Da die Menge $ \mbox{$\{(x,y)^\text{t}\in\mathbb{R}^2\,\vert\,x^2+y^2\le 1\}$}$ kompakt ist, und da $ \mbox{$f$}$ stetig ist, existieren dort sowohl globales Maximum als auch globales Minimum. Nach dem obigen Resultat werden beide auf dem Rand $ \mbox{$\{(x,y)^\text{t}\in\mathbb{R}^2\,\vert\,x^2+y^2=1\}$}$ angenommen.

Die Nebenbedingung lautet mit $ \mbox{$g(x,y) := x^2+y^2-1$}$ also $ \mbox{$g(x,y) = 0$}$ . Sei

$ \mbox{$\displaystyle
F(x,y)\; :=\; f(x,y) - \lambda g(x,y) \; .
$}$

Wir wollen die Multiplikatorenregel von Lagrange verwenden. Die dabei notwendige Voraussetzung

$ \mbox{$\displaystyle
\text{Rang} g'(x,y) \;=\; \text{Rang} (2x,2y) \;\overset{\!}{=}\; 1
$}$
ist erfüllt für alle $ \mbox{$(x,y)^\text{t}\in\mathbb{R}^2$}$ mit $ \mbox{$g(x,y) = 0$}$ .

Wir bestimmen nun alle regulären kritischen Punkte unter Nebenbedingung $ \mbox{$g = 0$}$ vermittels

$ \mbox{$\displaystyle
F'(x,y)\;=\;((y+1)e^{x(y+1)},xe^{x(y+1)})^\text{t} - \lambda (2x,2y)^\text{t} \;\overset{\!}{=}\; 0\; ,
$}$
d.h. vermittels
$ \mbox{$\displaystyle
\begin{array}{rcl}
(y+1)e^{x(y+1)} & = & 2\lambda x\\
xe^{x(y+1)} & = & 2\lambda y \; . \\
\end{array}$}$

Im Falle $ \mbox{$x=0$}$ folgt aus der Nebenbedingung $ \mbox{$y=\pm 1$}$ , und somit $ \mbox{$\lambda = 0$}$ . Nun gibt $ \mbox{$y= +1$}$ einen Widerspruch. Dagegen ist $ \mbox{$(0,-1)^\text{t}$}$ in der Tat ein kritischer Punkt.

Im Falle $ \mbox{$y=0$}$ ist gemäß der Nebenbedingung $ \mbox{$x=\pm 1$}$ , im Widerspruch zur obigen Gleichung.

Daher können wir uns von nun an auf den Fall $ \mbox{$x\ne 0$}$ und $ \mbox{$y\ne 0$}$ konzentrieren. Es ergibt sich

$ \mbox{$\displaystyle
\begin{array}{rcl}
\dfrac{y+1}{x} & = & 2\lambda e^{-x(y...
...vspace*{2mm}\\
\dfrac{x}{y} & = & 2\lambda e^{-x(y+1)}\; . \\
\end{array}$}$
Gleichsetzen der linken Seiten gibt $ \mbox{$\dfrac{x}{y} = \dfrac{y+1}{x}$}$ , d.h. $ \mbox{$x^2 = y^2 + y$}$ . Unter Verwendung der Nebenbedingung $ \mbox{$x^2+y^2=1$}$ ergibt sich
$ \mbox{$\displaystyle
2y^2+y-1\;=\; 0 \; ,
$}$
und hieraus $ \mbox{$y\in\{ -1, 1/2 \}$}$ . Insgesamt haben wir also die kritischen Punkte
$ \mbox{$\displaystyle
(0,-1)^\text{t}\;,\;\;\; \left(-\dfrac{\sqrt 3}{2},\dfra...
...t)^\text{t}\; ,\;\;\; \left(\dfrac{\sqrt 3}{2},\dfrac{1}{2}\right)^\text{t}
$}$
für lokale Extremstellen unter der Nebenbedingung $ \mbox{$g = 0$}$ .

Wir vergleichen die Funktionswerte

$ \mbox{$\displaystyle
f(0,-1)=1,\quad f(-\tfrac{\sqrt 3}{2},\tfrac{1}{2})=e^{-...
... 3}{4}}, \quad
f(\tfrac{\sqrt 3}{2},\tfrac{1}{2})=e^{\frac{3\sqrt 3}{4}}\; .
$}$
Also hat $ \mbox{$f$}$ unter der Nebenbedingung $ \mbox{$g = 0$}$ ein globales Maximum an der Stelle $ \mbox{$(\frac{\sqrt 3}{2},\frac{1}{2})^\text{t}$}$ mit dem Funktionswert $ \mbox{$e^{\frac{3\sqrt 3}{4}}$}$ und ein globales Minimum an der Stelle $ \mbox{$(-\frac{\sqrt 3}{2},\frac{1}{2})^\text{t}$}$ mit dem Funktionswert $ \mbox{$e^{-\frac{3\sqrt 3}{4}}$}$ .

Wie bereits oben ausgeführt, sind dies auch die globalen Extrema von $ \mbox{$f$}$ auf der Menge $ \mbox{$\{(x,y)^\text{t}\in\mathbb{R}^2 :x^2+y^2\leq 1\}$}$ .

Untersuchen wir unsere kritischen Punkte interessehalber auch noch auf lokale Extrema unter Nebenbedingung $ \mbox{$g = 0$}$ . Natürlich wissen wir bereits, daß die beiden globalen Extrema auch lokale Extrema zu sein haben. Mit etwas Glück - unsere diesbezügliche Bedingung ist zwar hinreichend, nicht aber notwendig - sollte sich das auch bestätigen.

Für beliebiges $ \mbox{$\lambda$}$ und beliebiges $ \mbox{$x$}$ wird $ \mbox{$\text{H}_F(x) = e^{x(y+1)}\begin{pmatrix}(y+1)^2-2\lambda&\; 1 + x + xy\;\\  \; 1 + x + xy\;&x^2-2\lambda\end{pmatrix}$}$ .

Bei $ \mbox{$(\frac{\sqrt 3}{2},\frac{1}{2})^\text{t}$}$ ist $ \mbox{$\lambda = \sqrt{3} e^{\frac{3\sqrt 3}{4}}$}$ , es ist $ \mbox{$g'(\frac{\sqrt 3}{2},\frac{1}{2}) = (\sqrt{3}, 1)$}$ , und wir können $ \mbox{$\text{T}_g(\frac{\sqrt 3}{2},\frac{1}{2}) =
\begin{pmatrix}1\\  -\sqrt{3}\end{pmatrix}$}$ nehmen. Die relative Hessematrix ergibt sich zu

$ \mbox{$\displaystyle
\text{H}_{f;g}(\frac{\sqrt 3}{2},\frac{1}{2})
\;=\; e^{...
...x}\;=\; e^{\frac{3\sqrt 3}{4}} (- 10\sqrt{3})\; \in\; \mathbb{R}^{1\times 1}
$}$
und ist negativ definit. In der Tat liegt also bei $ \mbox{$(\frac{\sqrt 3}{2},\frac{1}{2})^\text{t}$}$ ein lokales Maximum vor.

Bei $ \mbox{$(-\frac{\sqrt 3}{2},\frac{1}{2})^\text{t}$}$ ist $ \mbox{$\lambda = -\sqrt{3} e^{\frac{3\sqrt 3}{4}}$}$ , es ist $ \mbox{$g'(-\frac{\sqrt 3}{2},\frac{1}{2}) = (-\sqrt{3}, 1)$}$ , und wir können $ \mbox{$\text{T}_g(\frac{\sqrt 3}{2},\frac{1}{2}) =
\begin{pmatrix}1\\  \sqrt{3}\end{pmatrix}$}$ nehmen. Die relative Hessematrix ergibt sich zu

$ \mbox{$\displaystyle
\text{H}_{f;g}(\frac{\sqrt 3}{2},\frac{1}{2})
\;=\; e^{...
...rix}\;=\; e^{\frac{3\sqrt 3}{4}} (10\sqrt{3})\; \in\; \mathbb{R}^{1\times 1}
$}$
und ist positiv definit. In der Tat liegt also bei $ \mbox{$(-\frac{\sqrt 3}{2},\frac{1}{2})^\text{t}$}$ ein lokales Minimum vor.

Bei $ \mbox{$(0,-1)$}$ ist $ \mbox{$\lambda = 0$}$ , es ist $ \mbox{$g'(0,-1) = (0, -2)$}$ , und wir können $ \mbox{$\text{T}_g(0,-1) = \begin{pmatrix}1 \\  0\end{pmatrix}$}$ nehmen. Die relative Hessematrix ergibt sich zu

$ \mbox{$\displaystyle
\text{H}_{f;g}(0,-1)
\;=\; \begin{pmatrix}1 & 0\end{pma...
...gin{pmatrix}1 \\  0\end{pmatrix}\;=\; (0)\; \in\; \mathbb{R}^{1\times 1}\; ,
$}$
und wir können im einzigen noch offengebliebenen Fall keine Entscheidung über ein lokales Extremum unter Nebenbedingung $ \mbox{$g = 0$}$ treffen.

Skizze von $ \mbox{$f$}$ auf $ \mbox{$\{(x,y)^\text{t}\in\mathbb{R}^2\,\vert\,x^2+y^2\le 1\}$}$ .

\includegraphics[width = 8cm]{s3.eps}